Proving Horizontal Tangents of y=Cos(x) & y=Sec(x) at x=0

  • Thread starter Thread starter Robokapp
  • Start date Start date
  • Tags Tags
    Derivatives Trig
Click For Summary
To prove that y=Cos(x) and y=Sec(x) have horizontal tangents at x=0, the derivatives are calculated. The derivative of y=Sec(x) is y' = sec(x)tan(x). At x=0, sec(0) equals 1 and tan(0) equals 0, resulting in y' = 1*0 = 0, confirming a horizontal tangent. The confusion arose from attempting to set sec(x)tan(x) equal to zero without recognizing that the product is zero when either factor is zero. Understanding that sec(0) is 1 and tan(0) is 0 clarifies the proof of the horizontal tangent for y=Sec(x).
Robokapp
Messages
218
Reaction score
0
im supposed to prove that

y=Cos(x)

and

y=Sec(x)

have horisontal tangents for x=0

i got the derivatives just fine, i proved the first on e with no problem, but what do i do with second? i get to a point where i must prove that Sec(x)Tan(x)=0 but sec x = 1/cos(x) and i can't work with that...i mean how can i divide 1 by something to get zero? it's an asymptote...so what do i do?

i know how to get derivatives, i don't know how to prove that the second one is having the horisontal tangent.
 
Physics news on Phys.org
y = secx
y' = secx.tanx = sinx/cos²x

at x = 0, sinx = 0, cosx = 1

y' = 0/1² = 0
==========
 
Fermat said:
y = secx
y' = secx.tanx = sinx/cos²x

at x = 0, sinx = 0, cosx = 1

y' = 0/1² = 0
==========

okay...i thought that when you have secx*tanx=0 you have to set them like when you have parenthesis...each one at a time. That was probably a better explanation than most teachers would give. Thank you.

What i was trying to do is prove that either tan or sec is some value and the second one is zero, so zero*value=0 but it doesn't work that way.

can i ask though...why was i wrong? i mean if you do (x)(X+1)=0 you're defenetly not wrong to set either one equl to zero...why wasn't my way working? (To be honest I am more interested on what went wrong than what the answer is...i can always copy that from the book but i don't want to).
 
As far as I can see, you weren't wrong!

You had,

y' = secx*tanx

at x = 0, secx= 1/cosx = 1/1 = 1, and tanx = 0,

So,

y' = 1*0 = 0

Waht you have is secx = 1 and the other one, tanx = 0
 
Question: A clock's minute hand has length 4 and its hour hand has length 3. What is the distance between the tips at the moment when it is increasing most rapidly?(Putnam Exam Question) Answer: Making assumption that both the hands moves at constant angular velocities, the answer is ## \sqrt{7} .## But don't you think this assumption is somewhat doubtful and wrong?

Similar threads

  • · Replies 10 ·
Replies
10
Views
2K
Replies
6
Views
2K
  • · Replies 8 ·
Replies
8
Views
2K
  • · Replies 1 ·
Replies
1
Views
974
  • · Replies 8 ·
Replies
8
Views
1K
Replies
5
Views
2K
  • · Replies 8 ·
Replies
8
Views
2K
Replies
4
Views
2K
  • · Replies 2 ·
Replies
2
Views
2K